PT3.S4.Q03

Jalaina1908Jalaina1908 Live Member
edited June 2022 in Logical Reasoning 17 karma

Unsure why E is correct. I chose B initially, then once I did BR I chose D. Please help!

Comments

  • luffyyyyluffyyyy Live Member
    634 karma

    For PT 3 S4 Q3, the answer is actually C and not E. Is it the parallel flaw question you are referring to ?

Sign In or Register to comment.